Prova ITA 2020 – Matemática – Resolução Comentada

Prova ITA 2020 – Matemática – Resolução Comentada

Fala, pessoal… Tudo bem? Sou o prof. Victor So, do Estratégia Vestibulares e Carreiras Militares. Neste artigo, você vai conferir a resolução das questões da prova de Matemática do ITA 2020. Também deixei disponível a correção em PDF. Assim, você vai poder baixar gratuitamente. Vamos nessa??

">Matemática – Prova ITA 2020 – Resolvida" class="wp-block-file__button" download>Baixar

Prova ITA 2020

Sejam \large x_1,x_2,x_3,x_4,x_5,x_6 números reais tais que \large 2^{x_1}=4;3^{x_2}=5;4^{x_3}=6;5^{x_4}=7;6^{x_5}=8 e \large 7^{x_6}=9. Então, o produto \large x_1x_2x_3x_4x_5x_6 é igual a:

a) 6.

b) 8.

c) 10.

d) 12.

e) 14.

Resolução Comentada

Os números reais podem ser escritos como:

\large 2^{x_1}=4\Rightarrow x_1=\log_2{4}

\large 3^{x_2}=5\Rightarrow x_2=\log_3{5}

\large 4^{x_3}=6\Rightarrow x_3=\log_4{6}

\large 5^{x_4}=7\Rightarrow x_4=\log_5{7}

\large 6^{x_5}=8\Rightarrow x_5=\log_6{8}

\large 7^{x_6}=9\Rightarrow x_6=\log_7{9}

Assim, fazendo o produto entre eles, obtemos:

\large x_1x_2x_3x_4x_5x_6=\log_2{4}\cdot\log_3{5}\cdot\log_4{6}\cdot\log_5{7}\cdot\log_6{8}\cdot\log_7{9}

Podemos usar a seguinte propriedade dos logaritmos para simplificar a expressão:

\large \log_b{a}\cdot\log_a{c}=\log_b{c}

Reorganizando os termos da expressão:

\large \log_2{4}\cdot\log_4{6}\cdot\log_3{5}\cdot\log_5{7}\cdot\log_7{9}\cdot\log_6{8}=

\large \log_2{6}\cdot\log_6{8}\cdot\log_3{7}\cdot\log_7{9}=

\large \log_2{8}\cdot\log_3{9}=\log_2{2^3}\cdot\log_3{3^2}=3\cdot2=6

\large \therefore x_1x_2x_3x_4x_5x_6=6

Gabarito: A

Questão 42

Sejam \large a, \large b e \large c números reais, \large a\neq 0, tais que \large a^2+b^2=c^2. Se \large a, \large b e \large c formam, nessa ordem, uma progressão geométrica de razão \large k, então o produto \large P e a soma \large S de todos os possíveis valores para \large k são iguais a:

a) \large P=1 e \large S=0

b) \large P=-1 e \large S=1

c) \large P=-1 e \large S=-1

d) \large P=\frac{-\left(1+\sqrt5\right)}{2} e \large S=0

e) \large P=\frac{\left(1+\sqrt5\right)^2}{4} e \large S=0

Resolução Comentada

Vamos reescrever a sequência \large (a,\ b,\ c) como \large \left(\frac{b}{k},\ b,\ bk\right) para simplificar as contas.

Assim, a partir do enunciado, podemos escrever:

\large \left(\frac{b}{k}\right)^2+b^2={(bk)}^2

Como \large a\neq0 , implica que \large b\neq0

Dessa forma, temos: \large \left(\frac{1}{k}\right)^2+1=k^2

\large k^4-k^2-1=0

Façamos \large y=k^2

\large y^2-y-1=0

\large y=k^2=\frac{1+\sqrt5}{2} ou \large \ y=k^2=\frac{1-\sqrt5}{2}

Como k é um número real, temos que

\large k^2=\frac{1+\sqrt5}{2}\Rightarrow k=\pm\sqrt{\frac{1+\sqrt5}{2}}

Assim, \large P=-\frac{1+\sqrt5}{2} e \large S=0

Gabarito: D

Questão 43

A parte real da soma infinita da progressão geométrica cujo termo geral \large a_n é dado por

\large a_n=\frac{\cos{n}+i\cdot s e n\ n}{2^n},n=1,\ 2,\ 3,\ \ldots

é igual a

a) \large \frac{-1+2\cos{1}}{5-4\cos{1}}

b) \large \frac{-2+4\cos{1}}{5-4\cos{1}}

c) \large \frac{4-2\cos{1}}{5-4\cos{1}}

d) \large \frac{1+2\cos{1}}{5-4\cos{1}}

e) \large \frac{2+4\cos{1}}{5-4\cos{1}}

Resolução Comentada

Note que podemos escrever o termo geral da seguinte forma:

\large a_n=\frac{cis\ n}{2^n}=\frac{\left(cis\ 1\right)^n}{2^n}\Rightarrow a_n=\left(\frac{cis\ 1}{2}\right)^n

Usando o termo geral, temos a seguinte sequência:

\large \left(\frac{cis\ 1}{2},\left(\frac{cis\ 1}{2}\right)^2,\left(\frac{cis\ 1}{2}\right)^3,\ \ldots,\left(\frac{cis\ 1}{2}\right)^n,\ldots\right)

Logo, a razão da PG é:

\large q=\frac{cis\ 1}{2}

Como \large \left|q\right|=\frac{1}{2}<1, temos que a soma infinita converge. Assim, usando a fórmula da soma infinita da PG, obtemos:

\large S=\frac{a_1}{1-q}=\frac{\frac{cis\ 1}{2}}{1-\frac{cis\ 1}{2}}=\frac{\frac{cis\ 1}{2}}{\frac{2-cis\ 1}{2}}=\frac{cis\ 1}{2-cis\ 1}

\large \Rightarrow S=\frac{\cos{1}+i\ sen\ 1}{2-\cos{1}-isen\ 1}

Multiplicando o numerador e o denominador pelo conjugado do denominador, temos:

\large S=\frac{\left(\cos{1}+i\ sen\ 1\right)}{\left(2-\cos{1}-isen\ 1\right)}\cdot\frac{\left(2-\cos{1}+isen\ 1\right)}{\left(2-\cos{1}+isen\ 1\right)}

\large S=\frac{\cos{1}\left(2-\cos{1}\right)+isen\ 1\cos{1}+i\ sen\ 1\left(2-\cos{1}\right)-sen^21}{\left(2-\cos{1}\right)^2+sen^21}

\large S=\frac{\cos{1}\left(2-\cos{1}\right)-sen^21+i\left[sen\ 1\cos{1}+isen\ 1\left(2-\cos{1}\right)\right]}{\left(2-\cos{1}\right)^2+sen^21}

A parte real da soma infinita é dada por:

\large S=\frac{\cos{1}\left(2-\cos{1}\right)-sen^21}{\left(2-\cos{1}\right)^2+sen^21}

Simplificando a expressão:

\large S=\frac{2\cos{1}-\cos^2{1}-sen^21}{4-4\cos{1}+\cos^2{1}+sen^21}=\frac{2\cos{1}-1}{4-4\cos{1}+1}

\large \therefore S=\frac{-1+2\cos{1}}{5-4\cos{1}}

Gabarito: A

Questão 44

Duas curvas planas \large c_{1} e \large c_{2} são definidas pelas equações

\large c_1:16x^2+9y^2-224x-72y+640=0,

\large c_2:x^2+y^2+4x-10y+13=0.

Sejam \large P e \large Q os pontos de interseção de \large c_1 como o eixo \large x e \large R e \large S os pontos de interseção de \large c_{2} como o eixo \large y. A área do quadrilátero convexo de vértices P, Q, R e S é igual a:

a) \large 15+7\sqrt3

b) \large 15-7\sqrt3

c) \large 15+14\sqrt3

d) \large 15-14\sqrt3

e) \large 25+10\sqrt3

Resolução Comentada

Inicialmente, devemos encontrar as coordenadas dos pontos P, Q, R, S. Como P e Q são os pontos de interseção de \large c_{1} como eixo x, temos:

\large P=\left(x_p,0\right) e \large Q=\left(x_q,0\right)

Fazendo \large y=0 na equação de \large c_1

\large 16x^2-224x+640=0

\large \Rightarrow x^2-14x+40=0

\large \Rightarrow\left(x-10\right)\left(x-4\right)=0

Portanto, as raízes são \large x=10 ou \large x=4.

Assim, temos os pontos \large P\left(4,\ 0\right) e \large Q\left(10,\ 0\right).

Resta encontrar R e S. Como esses pontos são a interseção de \large c_2 como o eixo y, temos \large R=\left(0,\ y_R\right) e \large S=\left(0,\ y_S\right). Fazendo \large x=0 em \large c_2:

\large y^2-10y+13=0

\large \Rightarrow y=\frac{10\pm\sqrt{48}}{2}=\frac{10\pm4\sqrt3}{2}=5\pm2\sqrt3

Assim, temos \large R=\left(0,\ 5+2\sqrt3\right) e \large S=\left(0,\ 5-2\sqrt3\right).

Esboçando os pontos no plano cartesiano, temos a seguinte figura:

A área pedida é dada por:

\large \left[PQRS\right]=\left[QRO\right]-\left[PSO\right]=

\large \frac{1}{2}\cdot10\cdot\left(5+2\sqrt3\right)-\frac{1}{2}\cdot4\cdot\left(5-2\sqrt3\right)

\large \left[PQRS\right]=25+10\sqrt3-10+4\sqrt3=

\large 15+14\sqrt3

Gabarito: C

Questão 45

A cada aniversário, seu bolo tem uma quantidade de velas igual à sua idade. As velas são vendidas em pacotes com 12 unidades e todo ano é comprado apenas um novo pacote. As velas remanescentes são guardadas para os anos seguintes, desde o seu primeiro aniversário. Qual a sua idade, em anos, no primeiro ano em que as velas serão insuficientes?

a) 12.

b) 23.

c) 24.

d) 36.

e) 38.

Resolução Comentada

Veja que a quantidade de velas gastas a cada aniversário pode ser vista como uma progressão aritmética de razão 1.

Assim, o total de velas gastas até o n-ésimo aniversário é:

\large V_T=\frac{\left(1+n\right)n}{2}

Como todo ano um novo pacote de 12 velas é compradas, temos até o n-ésimo aniversário:

\large 12n-\frac{\left(1+n\right)n}{2} velas remanescentes

O primeiro ano em que as velas serão insuficientes ocorrerá quando as velas remanescentes satisfazerem a condição:

\large 12n-\frac{\left(1+n\right)n}{2}<0\Rightarrow12n<\frac{\left(1+n\right)n}{2}

Sendo n a idade, temos que \large n\neq0, logo:

\large n=24.

Gabarito: C

Questão 46

Seja A um ponto externo a circunferência \large \lambda de centro O e raio r. Considere uma reta passando por A e secante a \large \lambda nos pontos C e D tal que o segmento AC é o externo a \large \lambda e tem comprimento igual a r. Seja B o ponto de \large \lambda tal que O pertence ao segmento AB. Se o ângulo \large B\hat{A}D mede 10°, então a medida do ângulo \large B\hat{O}D é igual a:

a) 25°.

b) 30°.

c) 35°.

d) 40°.

e) 45°.

Resolução Comentada

De acordo com o enunciado, temos a seguinte figura:

Queremos determinar \large \alpha. Perceba que OCA é um triângulo isósceles, pois \large CO=CA=r. Logo:

Como \large \beta é ângulo externo ao \large \Delta OCA, então \large \beta=10\degree+10\degree=20\degree. Sabendo que a soma dos ângulos internos de um triângulo deve ser 180°, temos:

\large C\hat{O}D+\beta+\beta=180\degree\Rightarrow

\large C\hat{O}D=180\degree-40\degree=140\degree

Assim, \large \alpha é dada por:

\large \alpha+140\degree+10\degree=180\degree\therefore \alpha =30\degree

Gabarito: B

Questão 47

Se \large \alpha um número real satisfazendo \large 0<a<\frac{\pi}{2}. Então, a soma de todos os valores de \large x\in\left[0,\ 2\pi\right] que satisfazem a equação

\large \cos{x}\ sen\left(a+x\right)=sen\ a

é igual a

a) \large 5\pi+2a

b) \large 5\pi+a

c) \large 5\pi

d) \large 5\pi-a

e) \large 5\pi-2a

Resolução Comentada

Reescrevendo a equação, obtemos:

\large \cos{x}\ sen\left(a+x\right)=sen\ a

\large \cos{x}\left(sena\cos{x}+senx\cos{a}\right)=sen\ a

\large sena\cos^2{x}+senx\cos{x}\cos{a}=sen\ a

Note que

\large sen\ 2x=2senx\cos{x}\Rightarrow senx\cos{x}=\frac{sen\ 2x}{2}

\large \cos{2x}=2\cos^2{x}-1\Rightarrow\cos^2{x}=\frac{1+\cos{2x}}{2}

Substituindo na equação:

\large sena\left(\frac{1+\cos{2x}}{2}\right)+\left(\frac{sen\ 2x}{2}\right)\cos{a}=sen\ a

\large \frac{sen\ a}{2}+\frac{sen\ a\cos{2x}}{2}+\frac{sen\ 2x\cos{a}}{2}=sen\ a

\large \frac{sen\ a\cos{2x}+sen\ 2x\cos{a}}{2}=\frac{sen\ a}{2}

\large sen\left(a+2x\right)=sen\ a

Assim, temos as seguintes soluções:

\large a+2x=a+2k\pi\Rightarrow x=k\pi,k\in\mathbb{Z}

ou

\large a+2x=\pi-a+2k\pi\Rightarrow

\large 2x=\pi-2a+2k\pi\Rightarrow

\large x=\frac{\pi}{2}-a+k\pi,k\in\mathbb{Z}

Para o intervalo \large x\in\left[0,\ 2\pi\right] e lembrando que \large x=k\pi\Rightarrow x\in\left\{0,\ \pi,\ 2\pi\right\}

\large x=\frac{\pi}{2}-a+k\pi\Rightarrow x\in\left\{\frac{\pi}{2}-a,\frac{3\pi}{2}-a\right\}

Somando-se as soluções:

\large S=0+\pi+2\pi+\frac{\pi}{2}-a+\frac{3\pi}{2}-a=5\pi-2a

Gabarito: E

Questão 48

Considere o polinômio \large p\left(x\right)=x^3-mx^2+x+5+n, sendo m,n números reais fixados. Sabe-se que toda raiz \large z=a+bi, com \large a,b\in\mathbb{R}, da equação \large p\left(z\right)=0 satisfaz a igualdade \large a=mb^2+nb-1. Então, a soma dos quadrados das raízes de \large p\left(z\right)=0 é igual a:

a) 6.

b) 7.

c) 8.

d) 9.

e) 10.

Resolução Comentada

Pelo teorema fundamental da álgebra, o polinômio possui 3 raízes. Além disso, como os coeficientes do polinômio são reais, se tivermos uma raiz complexa, pelo teorema da raiz complexa conjugada, podemos afirmar que o conjugado dessa raiz também é raiz. Assim, temos as seguintes possibilidades:

I) duas raízes complexas e uma real

II) três raízes reais

Para o caso II de apenas raízes reais, temos da condição do enunciado, que toda raiz \large z=a+bi satisfaz a igualdade \large a=mb^2+nb-1 , ou seja, as raízes reais implicam \large b=0. Logo, todas as raízes são:

\large z=a=m\left(0\right)^2+n\left(0\right)-1\Rightarrow z_1=z_2=z_3=-1

Aplicando a relação de Girard para a soma do produto dois a dois:

\large z_1z_2+z_1z_3+z_2z_3=1

Mas, como \large z_1=z_2=z_3=-1:

\large z_1z_2+z_1z_3+z_2z_3=

\large \left(-1\right)\left(-1\right)+\left(-1\right)\left(-1\right)+\left(-1\right)\left(-1\right)=3

Portanto, chegamos a um absurdo!

A única possibilidade é a I, duas raízes complexas conjugadas e uma real. Então, sejam as raízes, para \large p,q,r\in\mathbb{R}:

\large z_1=p+qi

\large z_2=p-qi

\large z_3=r

Da condição do enunciado:

\large a=mb^2+nb-1

\large z_1\Rightarrow p=mq^2+nq-1\ \ \left(eq.I\right)

\large z_2\Rightarrow p=mq^2-nq-1\ \ \left(eq.\ II\right)

Da \large eq.\ I e \large eq.\ II , temos \large nq=0, logo:

\large p=mq^2-1

\large nq=0

Se \large q=0, teremos raízes reais, portanto, \large n=0.

Como \large z_3=r, temos \large r=-1\therefore z_3=-1

O polinômio é:

\large p\left(x\right)=x^3-mx^2+x+5

Aplicando Girard:

\large z_1+z_2+z_3=m\Rightarrow

\large p+qi+p-qi-1=m\Rightarrow

\large \fbox{$2p=m+1\ \left(eq.\ III\right)$}

\large z_1z_2+z_1z_3+z_2z_3=1\Rightarrow

\large \left(p+qi\right)\left(p-qi\right)+\left(-1\right)\left(p+qi+p-qi\right)=1

\large \Rightarrow\fbox{$p^2+q^2=1+2p\ \left(eq.\ IV\right)$}

\large z_1z_2z_3=-5\Rightarrow\left(p+qi\right)\left(p-qi\right)\left(-1\right)=

\large -5\Rightarrow\fbox{$p^2+q^2=5\ \left(eq.\ V\right)$}

Usando a \large eq.V na \large eq.IV:

\large 5=1+2p\Rightarrow2p=4\therefore p=2

Substituindo \large p=2 na \large eq.IV:

\large 4+q^2=1+4\Rightarrow q=\pm1

Assim, a soma dos quadrados das raízes é:

\large S=z_1^2+z_2^2+z_3^2=

\large \left(p+qi\right)^2+\left(p-qi\right)^2+\left(-1\right)^2

\large S=p^2+2pqi-q^2+p^2-2pqi-q^2+1=

\large 2p^2-2q^2+1=2\left(2\right)^2-2\left(-1\right)^2+1

\large S=7

Gabarito: B

Questão 49

A expansão decimal do número \large 100!=100\cdot99\cdot\cdot\cdot2\cdot1 possui muitos algarismos iguais a zero. Contando da direita para a esquerda, a partir do dígito das unidades, o número de zeros, que esse número possui antes de um dígito não nulo aparecer, é igual a

a) 20.

b) 21.

c) 22.

d) 23.

e) 24.

Resolução Comentada

Seja a fatoração em primos, única pelo teorema fundamental da álgebra, de \large 100!:

\large 100!=2^a\cdot3^b\cdot5^c\cdot\ldots\cdot{97}^z

Mas \large 100!=1\cdot2\cdot3\cdot4\cdot5\cdot6\cdot\ldots\cdot98\cdot99\cdot100

Dado um número inteiro positivo N, a quantidade de zeros em seu final é igual ao número de vezes em que se pode dividir por 10 e continuar com um inteiro positivo. A cada divisão, diminui-se uma unidade dos expoentes de 2 e de 5. Logo, é possível dividir por \large 10\ min\left \{ a,c \right \} vezes.

Acontece que em m!, para todo inteiro positivo m, temos sempre que o expoente de 5 é menor ou igual ao expoente de 2, isto é, \large c\le a . Logo, \large min\left \{ a,c \right \}=c. O problema agora é descobrir o expoente de 5 em 100!

Contemos as contribuições de cada \large k\in \left \{ 1,2,..., 99,100 \right \}.

Cada múltiplo de 5 contribui com pelo menos um fator 5.

Cada múltiplo de \large 5^2=25 contribui com um fator 5 extra.

Não existem múltiplos de \large 5^l com \large l\geq3.

Temos \large \left\lfloor\frac{100}{5}\right\rfloor+\left\lfloor\frac{100}{25}\right\rfloor=\ 20+\ 4=24zeros no fim de 100!.

Gabarito: E

Questão 50

Seja \large p\left(x\right)=ax^4+bx^3+cx^2+dx+e um polinômio com coeficientes reais. Sabendo que:

I – \large p\left(x\right) é divisível por \large x^2-4;

II – a soma das raízes de \large p\left(x\right) é igual a 1;

III – o produto das raízes de \large p\left(x\right) é igual a 3;

IV – \large p\left(-1\right)=-\frac{15}{4}

então, \large p(1) é igual a

a) \large -\frac{17}{2}

b) \large -\frac{19}{4}

c) \large -\frac{3}{2}

d) \large \frac{9}{4}

e) \large \frac{9}{2}

Resolução Comentada

De cada afirmação, temos:

I) Como \large p\left(x\right) é divisível por \large x^2-4, temos:

\large p\left(x\right)=q\left(x\right)\left(x^2-4\right)

As raízes do polinômio \large x^2-4 são \large x^2-4=0\Rightarrow x=\pm2, desse modo:

\large p\left(2\right)=16a+8b+4c+2d+e=0

\large p\left(-2\right)=16a-8b+4c-2d+e=0

II) Por Girard:

\large x_1+x_2+x_3+x_4=-\frac{b}{a}=1\Rightarrow b=-a

III) Por Girard:

\large x_1x_2x_3x_4=\frac{e}{a}=3\Rightarrow e=3a

IV) Substituindo \large x=-1 no polinômio

\large p\left(-1\right)=a-b+c-d+e=-\frac{15}{4}

Para \large b=-a e \large e=3, temos o seguinte sistema:

\large \left\{\begin{matrix} 16a+8b+4c+2d+e=0\\ 16a-8b+4c-2d+e=0\\ a-b+c-d+e=-\frac{15}{4}\end{matrix}\right.\Rightarrow

\large \left\{\begin{matrix} 16a-8a+4c+2d+3a=0\\ 16a+8a+4c-2d+3a=0\\ a+a+c-d+3a=-\frac{15}{4}\end{matrix}\right.\Rightarrow

\large \left\{\begin{matrix} 11a+4c+2d=0\\ 27a+4c-2d=0\\ 5a+c-d=-\frac{15}{4}\end{matrix}\right.

Multiplicando a última equação por 2:

\large \left\{\begin{matrix} 11a+4c+2d=0\\ 27a+4c-2d=0\\ 10a+2c-2d=-\frac{15}{2}\end{matrix}\right.

Somando a primeira equação com a segunda e a primeira com a terceira:

\large \left\{\begin{matrix} 38a+8c=0\\ 21a+6c=-\frac{15}{2}\end{matrix}\right.\Rightarrow

\large \left\{\begin{matrix} 19a+4c=0\\ 7a+2c=-\frac{5}{2}\end{matrix}\right.\Rightarrow

\large \left\{\begin{matrix} 19a+4c=0\\ -14a-4c=5\end{matrix}\right.\Rightarrow

\large 5a=a\therefore a=1

\large \therefore b=-1 e \large e=3

\large 19a+4c=0\Rightarrow19+4c=0\therefore c=-\frac{19}{4}

\large 11a+4c+2d=0\Rightarrow

\large 11+4\left(-\frac{19}{4}\right)+2d=0\Rightarrow

\large -8+2d=0\therefore d=4

Queremos \large p(1), logo:

\large p\left(1\right)=a+b+c+d+e=

\large 1-1-\frac{19}{4}+4+3=

\large -\frac{19}{4}+7=\frac{-19+28}{4}=\frac{9}{4}

Gabarito: D

Questão 51

Os pontos \large B=\left(1,\ 1+6\sqrt2\right) e \large C=\left(1+6\sqrt2,\ 1\right) são vértices do triângulo isósceles ABC de base BC, contido no primeiro quadrante. Se o raio da circunferência inscrita no triângulo mede 3, então as coordenadas do vértice A são:

a) \large \left(7\sqrt2,\ 7\sqrt2\right)

b) \large \left(\sqrt2,\ \sqrt2\right)

c) \large \left(1+7\sqrt2,\ 1+7\sqrt2\right)

d) \large \left(1+\sqrt2,\ 1+\sqrt2\right)

e) \large \left(1+6\sqrt2,\ 1+6\sqrt2\right)

Resolução Comentada

Como ABC é um triângulo isósceles, então sua altura em relação ao vértice A também é mediatriz em relação à base BC. Temos a seguinte figura:

M é ponto médio de BC, logo:

\large M=\frac{B+C}{2}\Rightarrow M=

\large \left(\frac{1+1+6\sqrt2}{2};\frac{1+1+6\sqrt2}{2}\right)\Rightarrow

\large M=\left(1+3\sqrt2;1+3\sqrt2\right)

Como \large \overline{AM} é mediatriz, temos que ela é perpendicular à reta \large \overline{BC}, vamos encontrar seu coeficiente angular:

\large m_{BC}=\frac{y_B-y_C}{x_B-x_C}=\frac{1+6\sqrt2-1}{1-\left(1+6\sqrt2\right)}

\large =\frac{6\sqrt2}{-6\sqrt2}=-1

\large m_{BC}\cdot m_{AC}=-1\Rightarrow

\large \left(-1\right)\cdot m_{AC}=-1\therefore m_{AC}=1

Como \large x_M=y_M e \large m_{AC}=1, temos que a reta que passa por M e A é y=x, ou seja, as coordenadas de A são da forma

\large A=\left(a,a\right)

Vamos resolver o problema por geometria plana. Sabemos que \large r=3 é o raio da circunferência inscrita ao triângulo. Consideremos a seguinte figura:

Do triângulo retângulo BCD:

\large BC^2=\left(6\sqrt2\right)^2+\left(6\sqrt2\right)^2=72+72=144\therefore BC=12

Podemos calcular a área do \large \Delta ABC de duas formas:

\large \left[ABC\right]=\frac{1}{2}\cdot b\cdot h=p\cdot r\Rightarrow

\large \frac{1}{2}\cdot12\cdot h=\frac{\left(l+l+12\right)}{2}\cdot3\Rightarrow

\large 4h=2l+12\Rightarrow\fbox{$2h=l+6$}

Veja que pelo teorema de Pitágoras no \large \Delta ABM

\large l^2=h^2+6^2

Usando \large 2h=l+6\Rightarrow l=2h-6:

\large \left(2h-6\right)^2=h^2+6^2\Rightarrow4h^2-24h+36=

\large h^2+36\Rightarrow3h^2-24h=0

\large 3h\left(h-8\right)=0\therefore h=8

Podemos usar a seguinte figura para calcular as coordenadas de A:

\large a=1+3\sqrt2+8sen\ 45°=1+32+822=1+72

\large \therefore A=\left(1+7\sqrt2;1+7\sqrt2\right)

Gabarito: C

Questão 52

Dado a\in \mathbb{R}, defina p=a+a^2 e q=a+a^3 e considere as seguintes afirmações:

I. se p ou q é irracional, então a é irracional.
II. se p e q são racionais, então a é racional.
III. se q é irracional, então p é irracional.

É(são) VERDADEIRA(S)

a) apenas I.

b) apenas II.

c) apenas I e II.

d) apenas I e III.

e) todas.

Resolução Comentada

I. Temos da afirmação \left(p\in\mathbb{I}\right)\vee\left(q\in\mathbb{I}\right)\rightarrow a\in\mathbb{I}. Usando sua contrapositiva:

\sim\left(a\in\mathbb{I}\right)\rightarrow\sim\left[\left(p\in\mathbb{I}\right)\vee\left(q\in\mathbb{I}\right)\right]

\sim\left(a\in\mathbb{I}\right)\rightarrow\sim\left(p\in\mathbb{I}\right)\land\sim\left(q\in\mathbb{I}\right)

a\in\mathbb{Q}\rightarrow\left(p\in\mathbb{Q}\right)\land\left(q\in\mathbb{Q}\right)

Assim, temos que verificar se a\in\mathbb{Q} implica que p\in\mathbb{Q} e q\in\mathbb{Q}. Como a\in\mathbb{Q}, temos a^2\in\mathbb{Q} e a^3\in\mathbb{Q}, p=a+a^2\in\mathbb{Q} e q=a+a^3\in\mathbb{Q}. Portanto, afirmação verdadeira.

II. Multiplicando-se p por a, temos:

ap=a^2+a^3

Podemos escrever a^{2} e a^{3} como:

p=a+a^2\Rightarrow a^2=p-a

q=a+a^3\Rightarrow a^3=q-a

Substituindo em ap:

ap=p-a+q-a\Rightarrow ap+2a=

p+q\Rightarrow\left(p+2\right)a=p+q

Para p\neq-2:

a=\frac{p+q}{p+2}

Se p,q\in\mathbb{Q}, temos que \frac{p+q}{p+2}\in\mathbb{Q} , logo, a\in\mathbb{Q}.

Para p=-2:

-2=a+a^2\Rightarrow a^2+a+2=0\Rightarrow

\Delta=1^2-4\cdot1\cdot2=1-8=-7<0

Portanto, a\notin\mathbb{R}, logo, não é possível.

Concluímos que a afirmação é verdadeira.

III. Tomemos o seguinte contraexemplo:

a=\sqrt3-\frac{1}{2}

q=a\left(1+a^2\right)=\left(\sqrt3-\frac{1}{2}\right)\left(1+\left(\sqrt3-\frac{1}{2}\right)^2\right)

q=\left(\frac{2\sqrt3-1}{2}\right)\left(1+3+\frac{1}{4}-\sqrt3\right)=

\frac{\left(2\sqrt3-1\right)\left(17-4\sqrt3\right)}{8}=

\frac{34\sqrt3-17-24+4\sqrt3}{8}

q=\frac{38\sqrt3-41}{8}\in\mathbb{I}

p=a\left(1+a\right)=\left(\sqrt3-\frac{1}{2}\right)\left(1+\sqrt3-\frac{1}{2}\right)=

\left(\sqrt3-\frac{1}{2}\right)\left(\sqrt3+\frac{1}{2}\right)=3-\frac{1}{4}=\frac{11}{4}\in\mathbb{Q}

Portanto, afirmação falsa.

Gabarito: C

Questão 53

Considere as seguintes afirmações:

  • I. Todo poliedro formado por 16 faces quadrangulares possui exatamente 18 vértices e 32 arestas.
  • II. Em todo poliedro convexo que possui 10 faces e 16 arestas, a soma dos ângulos de todas as faces é igual a 2160°.
  • III. Existe um poliedro com 15 faces, 22 arestas e 9 vértices.

É(são) VERDADEIRA (S)

a) apenas I.                                                                     

b) apenas II.                                

c) apenas III.

d) apenas I e II.

e) apenas II e III.

Resolução Comentada

I. Cuidado quando a afirmação diz “todo poliedro”, pois podemos ter um poliedro côncavo que não possui esse número de vértices e arestas. Veja o contraexemplo:

Esse poliedro possui 16 faces quadrangulares, 32 arestas e 16 vértices. Portanto, afirmação falsa.

II. A afirmação diz que o poliedro convexo possui 10 faces e 16 arestas, logo F=10 e A=16. Pela relação de Euler, temos:

V-A+F=2

V-16+10=2\therefore V=8

A soma dos ângulos internos de um poliedro convexo é dada por:

S_{i}=360\degree \cdot (V-2)

S_i=360\degree\cdot(8-2)=360\degree \cdot6=2160\degree

Portanto, afirmação verdadeira.

III. Se existe um poliedro com tais características, devemos ter:

2A=3F_3+4F_4+5F_5+\ldots

Note que

3F_3+4F_4+5F_5+\ldots

\geq3F_3+3F_4+3F_5+\ldots

=3\left(F_3+F_4+F_5+\ldots\right)=

3F\Rightarrow2A\geq3F

Substituindo A=22 e F=15, temos:

2\cdot22\geq3\cdot 15\Rightarrow44\geq45\ \left(Absurdo!\right)

Portanto, afirmação falsa.

Gabarito: B

Questão 54

Considere as seguintes afirmações:

I. Sejam \pi_1, \pi_2, \pi_3 três planos distintos, e secantes dois a dois segundo as retas distintas, r, s e t. Se r\cap s\neq\emptyset.
II. As projeções ortogonais de duas retas paralelas r e s sobre um plano \pi são duas retas paralelas.
III. Para quaisquer retas r, s e t reversas duas a duas, existe uma reta u paralela à r e concorrente com s e com t.

É(são) VERDADEIRA(S)

a) apenas I.

b) apenas II.

c) apenas I e II.

d) apenas I e III.

e) nenhuma.

Resolução Comentada

I. Como r,s,t são as retas da interseção dos três planos distintos e secantes dois a dois, temos:

r\in\pi_1\cap\pi_2

s\in\pi_1\cap\pi_3

t\in\pi_2\cap\pi_3

Se r\cap s\neq\emptyset e sabendo que as retas são distintas (não podem ser coincidentes), temos:

r\cap s=\left\{P\right\}

Logo:

P\in r\Rightarrow P\in\pi_1\ e\ P\in\pi_2

P\in s\Rightarrow P\in\pi_1\ e\ P\in\pi_3

P\in\pi_2\ e\ P\in\pi_3\Rightarrow P\in t

Portanto, r\cap s\cap t=\left\{P\right\}\neq\emptyset. Verdadeira.

II – Podemos ter duas retas paralelas e perpendiculares a um mesmo plano, a projeção delas no plano será dois pontos. Portanto, falsa.

III. Vejamos o contra-exemplo:

Note que tomando-se os planos \large \pi_1//\pi_2//\pi_3 e as retas \large r\in\pi_1,\ s\in\pi_2,\ t\in\pi_3, não paralelas entre elas, temos que a reta u paralela à r não pode ser concorrente simultaneamente à S e à T. Portanto, falta.

Gabarito: A

Questão 55

Considere o conjunto \large M(n,k) de todas as matrizes quadradas ordem n x n, com exatamente k, elementos iguais a 1, e os demais iguais a 0 (zero). Escolhendo aleatoriamente matrizes \large L\in M\left(3,\ 1\right) e \large R\in M\left(4,\ 2\right), a probabilidade de que \large L^2=0 e \large R^2=0 é igual a:

a) \large \frac{1}{3}

b) \large \frac{1}{5}

c) \large \frac{4}{15}

d) \large \frac{1}{30}

e) \large \frac{29}{30}

Resolução Comentada

I) Analisemos as matrizes \large L\in M\left(3,\ 1\right). Como n=3 e k=1, temos uma matriz de ordem 3×3 e um único elemento igual a 1. Temos, por exemplo:

\large L=\left(\begin{matrix}0&0&0\\1&0&0\\0&0&0\\\end{matrix}\right)\Rightarrow L^2=\left(\begin{matrix}0&0&0\\1&0&0\\0&0&0\\\end{matrix}\right)\left(\begin{matrix}0&0&0\\1&0&0\\0&0&0\\\end{matrix}\right)=\left(\begin{matrix}0&0&0\\0&0&0\\0&0&0\\\end{matrix}\right)

\large L=\left(\begin{matrix}1&0&0\\0&0&0\\0&0&0\\\end{matrix}\right)\Rightarrow L^2=\left(\begin{matrix}1&0&0\\0&0&0\\0&0&0\\\end{matrix}\right)\left(\begin{matrix}1&0&0\\0&0&0\\0&0&0\\\end{matrix}\right)=\left(\begin{matrix}1&0&0\\0&0&0\\0&0&0\\\end{matrix}\right)

Note que nesse caso, se o elemento 1 estiver na diagonal principal, \large L^2\neq0. Portanto, das 9 posições possíveis, podemos escolher apenas 6 (excluindo-se a diagonal principal) para inserir o elemento 1. Assim, temos:

\large P\left(L^2=0\right)=\frac{6}{9}=\frac{2}{3}

II) \large R\in M\left(4,\ 2\right), temos matrizes de ordem 4×4 e 2 elementos 1. Como analisamos no item I, se tivermos um elemento na diagonal principal, a matriz \large R^2\neq0. Então, para o primeiro elemento 1, temos que ele pode escolher 12 das 16 posições possíveis (excluindo-se a diagonal principal), logo, essa probabilidade é

\large \frac{12}{16}=\frac{3}{4}

Para o segundo elemento 1, devemos analisar do seguinte modo. Seja R definido por \large \left(r_{ij}\right), assim, temos que os elementos de \large R^2 serão:

\large \left(R^2\right)_{ij}=\sum_{k=1}^{4}\underbrace{r_{ik}}_{varia\ coluna}\cdot \underbrace{r_{ik}}_{varia\ linha}

Então, para obtermos uma matriz nula desse produto, temos que o segundo 1 não pode ocupar a diagonal principal (4 casos) e também não pode ocupar as posições que fazem com que \large r_{ik}\cdot r_{kj}=1, isso ocorre quando a linha do segundo elemento 1 é a coluna do primeiro (3 casos excluindo-se a diagonal principal) e quando a coluna do segundo elemento 1 é a linha do primeiro elemento (2 casos), logo, das 15 posições possíveis para o segundo elemento, temos:

\large P\left(R^2=0\right)=\frac{3}{4}\cdot\frac{15-4-3-2}{15}=\frac{3}{4}\cdot\frac{6}{15}=\frac{3}{10}

Portanto, a probabilidade pedida é:

\large P\left(L^2=0\right)\cdot P\left(R^2=0\right)=\frac{2}{3}\cdot\frac{3}{10}=\frac{1}{5}

Gabarito: B

É isso, pessoal! Espero que tenham curtido a resolução da prova de Matemática da prova da 1ª Fase do Vestibular ITA 2020. Sigam-me nas redes sociais. Têm muitas dicas lá. Mande uma mensagem, caso tenha tido alguma dúvida. Abraços!

Instagram: @profvictorso

Facebook: profvictorso

CURSOS ITA

Você pode gostar também